PT62.S2.Q25 - university president: our pool of applicants

_kizilbash_kizilbash Member
edited November 2017 in Logical Reasoning 62 karma

Hey guys, I am having trouble understanding what is wrong with C. Even though I initially picked the right answer which is A I can't seem up eliminate C.

A) the proposed explanation for the decline in applications applies to this case
C) an increase in tuition and fees at the university would guarantee a larger applicant pool

the conclusion of the of this question is "so, if we want to increase the size of our applicant pool we need to raise our tuition and fees"

I just felt like A did a better job of capturing the relationship between the argument and the conclusion. Also even if C is negated it wouldn't destroy the argument because I can reply by saying "well it wouldn't guarantee but there is still a high possibility"

thoughts?
Thanks in advance

https://7sage.com/lsat_explanations/lsat-62-section-2-question-25/

Comments

  • akistotleakistotle Member 🍌🍌
    9382 karma

    https://7sage.com/lsat_explanations/lsat-62-section-2-question-25/

    The argument is appealing to a possibility. It has to be the case that "one possible explanation" is the actual explanation for the argument to be true. (C) is a strengthening answer choice but isn't a necessary assumption.

    [Argument]
    P: One possible explanation for the decrease in the number of applicants: tuition is too low
    ———
    C: To increase the size of the applicant pool → raise tuition and fees

    (C) does not have to be the case. (C) is saying: raise tuition and fees → increase the size of the applicant pool

  • _kizilbash_kizilbash Member
    62 karma

    ahh ok. So what if the stimulas didn't say "one possible explanation". Would A still be correct?

Sign In or Register to comment.